A person pulls on a 9 kg crate against a 22 Newton frictional force, using a rope attached to the center of the crate. If the The crate began with a speed of 1.5 m/s and speeded up to 2.7 m/s while being pulled a horizontal distance of 2.0 meters. What is the work in J done by the force applied by the rope on the crate

Answers

Answer 1

Answer:

uninstell this apps nobody will give u ans its happening to me alsoi was in exam hall i thought this app will give answer but no


Related Questions

A force of 15 N toward the WEST is applied to a 4.0 kg box. Another force of 42 N toward the EAST is also applied to the 4.0 kg box. The net force on the 4.0 kg box
is

Answers

Answer :

[tex]\implies F_1 < F_2[/tex]

[tex] \implies F_{net} = F_2 - F1[/tex]

[tex]\implies F_{net} = 42 -15[/tex]

[tex]\implies \underline{ \boxed{ F_{net} = 27 \: N}}[/tex]

The net force on the 4.0 kg box is 27 N towards EAST.

A spinning electron produces a(n)
a. element.
b. magnetic field.
c. proton.
d. piece of iron.

Answers

Answer:

A spinning electron produces a magnetic field.

Explains the fine structure of Hydrogen lines.

please help me .finish this paper​

Answers

Solution-1:-

[tex]\boxed{\sf \dfrac{10\times 1000}{60\times 60}}[/tex]

Solution:-2

[tex]\boxed{\sf Sodium\:and\:Potassium}[/tex]

Solution:-3

[tex]\boxed{\sf 320m}[/tex]

Solution:-4

[tex]\boxed{\sf Rough\:tiles\:are\:used\:in\:bathroom}[/tex]

Solution:-5

[tex]\boxed{\sf Mg_3N_2}[/tex]

Solution:-6

[tex]\boxed{\sf Grapes\:and\:Rambutan}[/tex]

Solution:-7

[tex]\boxed{\sf {}^{}_{}N}[/tex]

Solution:-8

[tex]\boxed{\sf Galactuse}[/tex]

Solution:-9

[tex]\boxed{\sf Y-X}[/tex]

Solution:-10

[tex]\boxed{\sf Cell\:wall}[/tex]

I need the help
Please. I’m terrible at physics

Answers

Answer:

Explanation:

so opposite and equal , right?    forces are. soooo..

528+52= 580 N    is the force that is being exerted up on the scale

Two 0.20-kg balls, moving at 4 m/s east, strike a wall. Ball A bounces backwards at the same speed. Ball B stops. Which statement correctly describes the change in momentum of the two balls?

a. |ΔpBl<|ΔPA|
b. |ΔpBl=|ΔPA|
c. |ΔpB|>|ΔPA|
d. ΔpB > ΔPA

Answers

Answer:

Option A

Explanation:

From the question we are told that:

Mass [tex]m=0.20kg[/tex]

Velocity [tex]v=4m/s[/tex]

Generally the equation for momentum for Ball A is mathematically given by

Initial Momentum

 [tex]M_{a1}=mV[/tex]

 [tex]M_{a1}=0.2*4[/tex]

 [tex]M_{a1}=0.8[/tex]

Final Momentum

 [tex]M_{a2}=-0.8kgm/s[/tex]

Therefore

 [tex]\triangle M_a=-1.6kgm/s[/tex]

Generally the equation for momentum for Ball B is mathematically given by

Initial Momentum

 [tex]M_{b1}=mV[/tex]

 [tex]M_{b1}=0.2*4[/tex]

 [tex]M_{b1}=0.8[/tex]

Final Momentum

 [tex]M_{b2}=-0 kgm/s[/tex]

Therefore

 [tex]|\triangle M_a|>|\triangle Mb|[/tex]

Option A

The same constant force is used to accelerate two carts of the same mass, initially at rest, on horizontal frictionless tracks. The force is applied to cart A for twice as long a time as it is applied to cart B. The work the force does on A is WA; that on B is WB. Which statement is correct?

a. WA = WB
b. WA = 2WB.
c. WA=4WB
d. WB= 2WA

Answers

Answer:

Option (c).

Explanation:

Let the mass of each cart is m and the force is F.

Time for cart A is 2t and for cart B is t.

Work done is given by the

W= force x displacement

As the distance is given by

S= u t +0.5 at^2

So, when the time is doubled the distance is four times.

So, WA = F x 4 S

WB = F x S

WA= 4 WB

A 2 nC point charge is embedded at the center of a nonconducting sphere of radius = 4.8 cm which has a charge of 1 nC distributed uniformly throughout its volume. What is the magnitude of the electric field at a point that is 2.4 cm from the center of the sphere? Write your answer in terms of kN/C.

Answers

Answer: [tex]3.32\times 10^4\ kN/C[/tex]

Explanation:

Given

Charge at the center of the sphere is [tex]q=2\ nC[/tex]

Charge distributed over the entire sphere [tex]Q=1\ nC[/tex]

Radius of sphere [tex]r=4.8\ cm[/tex]

Using Guass law

[tex]\Rightarrow \oint \vec{E}\cdot \vec{dA}=\dfrac{q_{enc}}{\epsilon_o}\\\\\Rightarrow E\cdot 4\pi r^2=\dfrac{1}{\epsilon}(q+\dfrac{Q}{4\pi R^2}\times 4\pi r^2)\\\\\Rightarrow E\cdot 4\pi r^2=\dfrac{1}{\epsilon}(q+Q\dfrac{r^2}{R^2})\\\\\Rightarrow E\cdot (2.4\times 10^{-2})^2=9\times 10^9(2+1\cdot \dfrac{1}{4})\times 10^{-9}\\\\\Rightarrow E=3.32\times 10^4\ kN/C[/tex]

A barge is hauled along a straight-line section of canal by two horses harnessed to tow ropes and walking along the tow paths on
either side of the canal. Each horse pulls with a force of 839 N at an angle of 15° with the centerline of the canal. Find the sum of these
two forces on the barge.
answer in ___kN

Answers

Answer:

1.621 kN

Explanation:

Since each horse pulls with a force of 839 N at an angle of 15° with the centerline of the canal, the horizontal component of the force due to the first horse along the canal is F= 839cos15° N and its vertical component is F' = 839sin15° N(it is positive since it is perpendicular to the centerline of the canal and points upwards).

The horizontal component of the force due to the second horse along the canal is f = 839cos15° N and its vertical component is f' = -839sin15° N (it is negative since it is perpendicular to the centerline of the canal and points downwards).

So, the resultant horizontal component of force R = F + f = 839cos15° N + 839cos15° N = 2(839cos15°) N = 2(839 × 0.9659) = 2 × 810.412 = 1620.82 N

So, the resultant vertical component of force R' = F' + f' = 839sin15° N + (-839sin15° N) = 839sin15° N - 839sin15° N = 0 N

The magnitude of the resultant force which is the sum of the two forces is R" = √(R² + R'²)

= √(R² + 0²)  (since R' = 0)

= √R²

= R  

= 1620.82 N

= 1.62082 kN

≅ 1.621 kN

So, the sum of these  two forces on the barge is 1.621 kN

A 59.0 kg bungee jumper jumps off a bridge and undergoes simple harmonic motion. If the period of oscillation is 0.250 mins, what is the spring constant (in N/m) of the bungee cord, assuming it has negligible mass compared to that of the jumper

Answers

Answer:

The spring constant of the spring is 10.3 N/m.

Explanation:

Given that,

Mass of a bungee jumper, m = 59 kg

The period of oscillation, T = 0.25 min = 15 sec

We need to find the spring constant of the bungee cord. We know that the period of oscillation is given by :

[tex]T=2\pi\sqrt{\dfrac{m}{k}}[/tex]

Where

k is the spring constant

[tex]T^2=4\pi^2\times \dfrac{m}{k}\\\\k=4\pi^2\times \dfrac{m}{T^2}\\\\k=4\pi^2\times \dfrac{59}{(15)^2}\\\\k=10.3\ N/m[/tex]

So, the spring constant of the spring is 10.3 N/m.

A thin stream of water flows vertically downward. The stream bends toward a positively charged object when it is placed near it. The positively charged object is then removed. What will happen to the same stream of water when a negatively charged object is placed nearit

Answers

Answer:

the water jet and the negative object attract

Explanation:

Let's analyze the situation. Water is a good conductor of electricity, so when an object with a positive charge is brought closer, a charge of the opposite sign is created, which is why the two objects attract each other. The charge created comes from the ground

When claiming the object, the charge is reduced to zero or the charge goes to earth since water is a good conductor of electricity, therefore when approaching an object with a negative charge, more charges go to earth and the ring is left with a positive charge and attracts it to the object.

In short, the water jet and the negative object attract

A weight suspended from a spring is seen to bob up and down over a distance of 20 cm triply each second. What is the period? What is the amplitude?

Answers

The period and the amplitude of the weight suspended from spring are 0.33 seconds and 10 centimeters, respectively.

1) The period is given by:

[tex] T = \frac{1}{f} [/tex]

Where:

f: is the frequency = 3 bob up and down each second = 3 s⁻¹ = 3 Hz

[tex] T = \frac{1}{f} = \frac{1}{3 Hz} = 0.33 s [/tex]

Hence, the period is 0.33 s.

2) The amplitude is the distance between the equilibrium position and the maximum position traveled by the spring. Since the spring is moving up and down over a distance of 20 cm, then the amplitude is:          

[tex] A = \frac{20 cm}{2} = 10 cm [/tex]  

Therefore, the amplitude is 10 cm.          

You can learn more about the period and amplitude here: https://brainly.com/question/15169209?referrer=searchResults

I hope it helps you!                    

Using the data provided below, calculate the corrected wavelength for a spectroscope reading of 6.32.

Spectroscope Readings Known Helium Wavelengths (nm)
3.65 388.9
4.75 486.6
4.90 501.6
5.65 587.6
6.45 667.8
7.00 706.5

Answers

Answer:

646.6 nm

Explanation:

Using the data given, we fit a linear model :

Spectroscope Readings Known Helium Wavelengths (nm)

3.65 388.9

4.75 486.6

4.90 501.6

5.65 587.6

6.45 667.8

7.00 706.5

Using technology like excel to fit the model, the regression model obtained is :

Y = 97.96582X + 27.48458

Where, y is the predicted or corrected wavelength value.

This mod could be used to calculate the corrected wavelength for a given Spectroscope value :

Given a Spectroscope value of 6.32 ; the corrected wavelength is obtained by replacing x in the equation by 6.32 and calculate y ;

Y = 97.96582X + 27.48458

x = 6.32

Y = 97.96582(6.32) + 27.48458

Y = 619.1439824 + 27.48458

Y = 646.6285624

Corrected wavelength value is : 646.6

For a solid uniformly charged sphere of radius R, calculate the electric field at a distance R/2 outside the sphere, divided by the electric field at a distance R/2 inside the sphere. a) 9/8 b) infinity c) 2.0 d) 8.0 e) 8/9

Answers

Answer:

e

Explanation:

From the given information:

Suppose Q = total charge of the sphere.

here, the electric field outside the sphere at distance R/2 can be expressed as:

[tex]E_1 = \dfrac{1}{4 \pi \varepsilon _o}* \dfrac{Q}{(R + \dfrac{R}{2})^2}[/tex]

where:

[tex]k = \dfrac{1}{4 \pi \varepsilon _o}[/tex]

[tex]E_1 = \dfrac{kQ}{(\dfrac{3R}{2})^2}[/tex]

[tex]E_1 = \dfrac{4kQ}{9R^2}[/tex]

For the electric field inside the sphere, we have:

[tex]E_2 = \dfrac{kQr}{R^3}[/tex]

here:

r = distance of the point from the center = R/2

R = radius of the sphere

[tex]E_2 = \dfrac{kQ * \dfrac{R}{2}}{R^3}[/tex]

[tex]E_2 = \dfrac{kQ }{2R^2}[/tex]

As such, the ratio of the electric field outside the sphere to the one inside is:

[tex]\dfrac{E_1}{E_2} = \dfrac{ \dfrac{4kQ}{9R^2}}{ \dfrac{kQ }{2R^2}}[/tex]

[tex]\dfrac{E_1}{E_2} = \dfrac{4kQ}{9R^2} \times \dfrac{ 2R^2 }{kQ}[/tex]

[tex]\mathbf{\dfrac{E_1}{E_2} = \dfrac{8}{9}}[/tex]

For a solid uniformly charged sphere of radius R, the electric field at a distance R/2 outside the sphere, divided by the electric field at a distance R/2 inside the sphere is -  e) 8/9

Electric field due to a sphere at a distance R/2 outside the sphere can be calculated by assuming the sphere as a point mass

field  [tex]E2= \frac{k(Q)}{(3R/2)^2}[/tex]

=  [tex]\frac{4}{9} \frac{k(Q)}{R^2}[/tex]

The electric field at a distance R/2 inside the sphere E1= k(Q)(r)/R^3 where r is the distance from the center  [tex]E1= \frac{k(Q)}{(3R/2)^3}[/tex]

=

Thus, E2/E1

= (4/9)/(1/2)

= 8/9

Thus, the electric field at a distance R/2 outside the sphere, divided by the electric field at a distance R/2 inside the sphere is -  8/9

Learn more:

https://brainly.com/question/16908008

12. A car travels in a straight line with an average velocity of 80 km/h for 2.5h and then an average velocity of 40 km/h from 1.5 h (a) what is the total displacement for the 4 h trip? (b) What is the average velocity for the total trip?

Answers

Answer:

a. Total displacement = 140 km/h

b. Average velocity = 35 km/h

Explanation:

Given the following data;

Average velocity A = 80 km/h

Time A = 2.5 hours

Average velocity B = 40 km/h

Time B = 1.5 hours

a. To find the total displacement for the 4 h trip;

Total time = Time A + Time B

Total time = 2.5 + 1.5

Total time = 4 hours

Next, we would determine the displacement at each velocity.

Mathematically, displacement is given by the formula;

Displacement = velocity * time

Substituting into the formula, we have;

Displacement A = 80 * 2.5

Displacement A = 200 km/h

Displacement B = 40 * 1.5

Displacement B = 60 km/h

Total displacement = Displacement A - Displacement B

Total displacement = 200 - 60

Total displacement = 140 km/h

b. To find the average velocity for the total trip;

Mathematically, the average velocity of an object is given by the formula;

[tex] Average \; velocity = \frac {total \; displacement}{total \; time} [/tex]

Substituting into the formula, we have;

[tex] Average \; velocity = \frac {140}{4} [/tex]

Average velocity = 35 km/h

A covalent bond is formet by of electrons..?​

Answers

Answer:

The covalent bond is formed by pairs of electrons that are shared between two atom

Explanation:

The covalent bond is formed by pairs of electrons that are shared between two atoms, in general the electrons must have opposite spins to have a lower energy state.

In this bond, the electrons are between the two atoms and are shared between them in such a way that there is a configuration of eight electrons in the orbit.

HEELLPPPPPpppppppppppppppp

Answers

Explanation:

Given:

[tex]A_1[/tex] = 4.5 cm[tex]^2[/tex]

[tex]v_1[/tex] = 40 cm/s

[tex]v_2[/tex] = 90 cm/s

[tex]A_2[/tex] = ?

a) The continuity equation is given by

[tex]A_1v_1 = A_2v_2[/tex]

Solving for [tex]A_2[/tex],

[tex]A_2 = \dfrac{v_1}{v_2}A1 = \left(\dfrac{40\:\text{cm/s}}{90\:\text{cm/s}}\right)(4.5\:\text{cm}^2)[/tex]

[tex]= 2\:\text{cm}^2[/tex]

b) If the cross-sectional area is reduced by 50%, its new area [tex]A_2'[/tex] now is only 1 cm^2, which gives us a radius of

[tex]r = \sqrt{\dfrac{A_2'}{\pi}} = 0.564\:\text{cm}[/tex]

what is the prefix notation of 0.0000738?​

Answers

Answer:

The scientific notation of 738 is 7.38 x 1002.

I agree with the answer above me…


How does radiation from the sun spread throughout Earth's atmosphere?
- through convection currents
- through conduction currents
- through solar panels
- through hot water systems

Answers

Through conduction currents.
Followed by convection currents.

TRUE or FALSE: The acceleration of projectile is 0 m/s/s at the peak of the trajectory. Identify the evidence which supports your answer.

Answers

FALSE The vertical velocity of a projectile is 0 m/s at the peak of its trajectory; but the horizontal component of the velocity at the peak is whatever the value was when first launched

The vertical acceleration of the projectile is at 0 m/s while the horizontal acceleration of the projectile at the peak of the trajectory = initial acceleration  ( i.e. statement in the question is False )

Projectile motion follows a parabolic path with x and y components of its velocity and acceleration. also the acceleration of a projectile is subject only to the acceleration due to gravity unlike other kinds of motions.

In a parabolic motion an object ( projectile ) is thrown into the air and left to move through a parabolic path under the effect of acceleration due to gravity.

Hence we can conclude that the statement is false, because horizontal acceleration of the projectile at the peak of the trajectory = initial acceleration

Learn more : https://brainly.com/question/24658194

Object A has a mass m and a speed v, object B has a mass m/2 and a speed 4v, and object C has a mass 3m and a speed v/3. Rank the objects according to the magnitude of their momentum.

Required:
Rank from smallest to largest.

Answers

Answer:

Momentum of object A = Momentum of object C < momentum of B.

Explanation:

The momentum of an object is equal to the product of mass and velocity.

Object A has a mass m and a speed v. Its momentum is :

p = mv

Object B has a mass m/2 and a speed 4v. Its momentum is :

p = (m/2)×4v = 2mv

Object C has a mass 3m and a speed v/3. Its momentum is :

p = (3m)×(v/3) = mv

So,

Momentum of object A = Momentum of object C < momentum of B.

Which person ha the most freedom to make his or her own lifestyle decisions?
A. Frieda is 10 years old and lives with her grandmother
B. Vladimir is 3 years old and attends preschool
C. Quincy is 16 years old and lives in a dormitory
D. Lucinda is 32 years old and has two children

Answers

Answer:

C Quincy

Explanation:

Both Frieda and Vladimir are too young to be making their own decisions. Lucinda has limited freedom due to having two children, while Quincy is just now becoming an adult and has his whole life still ahead of him. Therefore, Quincy has the most freedom to make their own lifestyle decisions.

I hope this helps!

Answer:

The correct answer is C. Quincy is 16 years old and lives in a dormitory

Explanation:

Quincy being 16 may be able to get a job and hold a membership at a gym even if not at a gym he still has the most freedom period with Lucinda having children and Frieda and Vladimir being to young to make the choices of exercising on their own.

Please tell me if I'm wrong so I may give you the correct answer!

Happy Holidays!!  

Convert the unit of 0.0063 milliseconds into microseconds. (Answer in scientific notation)

Answers

Answer:

microsecond = 1 × 10-6 seconds

1 second = 1 × 100 seconds

1 microsecond = (1 / 1) × 10-6 × 10-0 seconds

1 microsecond = (1) × 10-6-0 seconds

1 microsecond = (1) × 10-6 seconds

1 microsecond = 1 × 1.0E-6 seconds

1 microsecond = 1.0E-6 seconds,just do like it

Answer:

6.3 x [tex]10^{-3}[/tex]

Explanation:

1 millisecond = 1000 microseconds

0.0063 millisecond = 6.3 microseconds

Una bala de 10 g se dispara contra un bloque de madera de 102 g inicialmente en reposo sobre una superficie horizontal. Después del impacto el bloque se desliza 8 m antes de detenerse. Si el coeficiente de fricción entre el bloque y la superficie es 0,5, ¿Cuál es la velocidad de la bala inmediatamente antes del impacto?

Answers

Answer:

una ess abola cola sola answer

An object carries a charge of -8.5 µC, while another carries a charge of -2.0 µC. How many electrons must be transferred from the first to the second object so that both objects have the same charge?

Answers

Answer:

Approximately 2*10^13 electrons must be transferred

Explanation:

Below is the given information:

First object carries charge = -8.5 µC

Number of electrons in 1st = 8.5 x 10^-6/(1.6 x 10^-19) = 5.3125 x 10^13

Second object carries a charge = -2.0 µC

The number of electrons in 2nd = 2*10^-6/(1.6*10^-19) = 1.25 x 10^13

so, approximately 2 x 10^13 electrons must be transferred

The _______ principle encourages us to resolve a set of stimuli, such as trees across a ridgeline, into smoothly flowing patterns
A.) depth perception.
B.) perception.
C.) similarity.
D.) continuity.

Answers

Answer:

The answer is continuity ( D )

Explanation:

PLZ MARK AS BRAINLIEST

The amount of charge that passes through the filament of a certain lightbulb in 2.00 s is 1.67 C. If the current is supplied by a 12.0 - V battery, what total energy in Joules is delivered to the lightbulb filament during 2.00 s

Answers

Answer:

E = 20.03 J

Explanation:

Given that,

The amount of charge that passes through the filament of a certain lightbulb in 2.00 s is 1.67 C,

Voltage, V = 12 V

We need to find the energy delivered to the lightbulb filament during 2.00 s.

The energy delivered is given by :

[tex]E=I^2Rt[/tex]. ....(1)

As,

[tex]I=\dfrac{q}{t}\\\\I=\dfrac{1.67}{2}\\\\I=0.835\ A[/tex]

As per Ohm's law, V = IR

[tex]R=\dfrac{V}{I}\\\\R=\dfrac{12}{0.835}\\\\R=14.37\ \Omega[/tex]

Using formula (1).

[tex]E=0.835^2\times 14.37\times 2\\\\=20.03\ J[/tex]

So, the energy delivered to the lightbulb filament is 20.03 J.

What is the strength of the electric field between two parallel conducting plates separated by 1.00 cm and having a potential difference (voltage) between them of 3.22 x 104 V

Answers

Answer:

[tex]E=3.22*10^6 N/C[/tex]

Explanation:

From the question we are told that:

Separation Distance [tex]d=1.0cm =0.01m[/tex]

Potential difference [tex]V=3.22 * 10^4 V[/tex]

Generally the equation for Electric Field strength is mathematically given by

 [tex]E=\frac{v}{d}[/tex]

 [tex]E=\frac{3.22*10^4}{0.01}[/tex]

 [tex]E=3.22*10^6 N/C[/tex]

A hungry monkey is sitting at the top of a tree 69 m above ground level. A person standing on the ground wants to feed the monkey. He uses a tee-shirt cannon to launch bananas at the monkey. If the person knows that the monkey is going to drop from the tree at the same instant that the person launches the bananas, how should the person aim the banana cannon

Answers

Answer:

Well if you want to be sure you should just throw it to the ground so then when he lands he can catch it.

If the cannon throws the banana with the same force the monkey falls

(m.g=Fz <=> m.9,81N/kg=...N).

Then the throw will slow down because of the gravitational pull.

Because the banana cannon is selfmade you can choose what mass the bananas in question have, so let that be the same as the monkeys.

The monkey falls with the speed of 9,81m.s => so it takes the monkey 7,1s to land.

If the cannon can shoot the banana at the same speed the monkey falls then they would cross in the middle.

So to do so you need to throw the bananas with a speed of at least 9,81m.s

Soo ... throw them with a force of that is greater then the gravitational pull and things will work out.

I'm sorry I don't know why I wrote all of this irrelevant information it's 2:21 right now and I'm tired.

kind regards

How many joules of energy are required to accelerate one kilogram of mass from rest to a velocity of 0.866c?

Answers

Answer:

the amount of energy needed is 1.8 x 10¹⁷ J.

Explanation:

Given;

mass of the object, m₀ = 1 kg

velocity of the object, v = 0.866 c

By physics convection, c is the speed of light = 3 x 10⁸ m/s

The energy needed is calculated as follows;

E = Mc²

As the object approaches the speed of light, the change in the mass of the object is given by Einstein's relativity formula;

[tex]M = \frac{M_0}{\sqrt{1- \frac{v^2}{c^2} } } \\\\ M = \frac{1}{\sqrt{1- \frac{(0.866c)^2}{c^2} } }\\\\ M = \frac{1}{\sqrt{1- \frac{0.74996c^2}{c^2} } }\\\\ M = \frac{1}{\sqrt{0.25} } \\\\ M = 2 \ kg[/tex]

The energy required is calculated as;

E = 2 x (3 x 10⁸)²

E = 1.8 x 10¹⁷ J

Therefore, the amount of energy needed is 1.8 x 10¹⁷ J.

A compact disk with a 12 cm diameter is rotating at 5.24 rad/s.

a. What is the linear speed _______m/s
b. What is the centripetal acceleration of a point on its outer rim _______
c. Consider a point on the CD that is halfway between its center and its outer rim. Without repeating all of the calculations required for parts (a) and (b), determine the linear speed of this point. _______m/s
d. Determine the centripetal acceleration of this point. _______

Answers

Answer:

(a) 31.44 m/s (b) 164.74 m/s²

Explanation:

Given that,

The diameter of a disk, d = 12 cm

Radius, r = 6 cm

Angular speed = 5.24 rad/s

(a) Linear speed,

[tex]v=r\omega\\\\v=6\times 5.24\\\\v=31.44\ m/s[/tex]

(b) Centripetal acceleration,

[tex]a=\dfrac{v^2}{r}\\\\a=\dfrac{31.44^2}{6}\\\\a=164.74\ m/s^2[/tex]

Other Questions
how would you detect sulphur in organic compounds write 10 multiple choice questions where you ask 3 respondents 10 questions about teenage pregnancy The perimeter of a rectangle is given by the formula P=2w+2l,where w is the width and l is the length.Rearrange the formula for the length ( ). Comment s'appelle la tendance qui prconise l'inspiration plurielle Katie, Merci de bien vouloir fermer mon compte, puisque je suis un voyou qui ne respecte pas le code thique Two sides of a right triangle measure 5 inches and 6 nches, and the third side measures 61 inches.The length of the third side is about inches _ . The perimeter of the triangle is about inches. __ Find the value of each expression.30 3 2 U (24 win Q7 A gardener needs to order fertiliser for a piece of land. The piece of land is a square with sides measuring 8 metres. This formula shows how many grams of fertiliser she needs. grams of fertiliser needed = length in metres x width in metres x 25 The supplier sells these bags of fertiliser. 10K 5kg 2kg 1kg 1 kilogram 2 kilograms 5 kilograms 10 kilograms Which bag of fertiliser should the gardener order? Include figures to explain your answer. Show all your working. What is indirect program memory addressing? Explain the working of the following instructions? a) JMP AXb) JMP LIST[DX]c) JMP NEAR PTR[DI+3] mAFD=90 . mAFB=31. Find mDFE.A. 87B. 29.5C. 31D. 28 Classify the following polynomials according to the number of terms. Combine any like terms first. Help question 25 pleasee Fatty acids are the building blocks of(1 point)monosaccharidesamino acidsDNAlipids Which layer of the atmosphere favours air transport?explain? [tex]\frac{3}{2x}[/tex]-[tex]\frac{11}{5}[/tex]=[tex]\frac{7}{8}[/tex].[tex]\frac{64}{49}[/tex] sequence 32,52,72,92 simplified The stock market crash triggered the beginning of the Great Depression the worst economic crisis in U.S history (which factor did not contribute to the crash) How do I view my work schedule Six Flags? This one is confusing guys pls tell me this answer as soon as possible